How to Write the AP Lang Argument Essay (With Example)

December 14, 2023

ap lang argument essay example

We’d like to let you in on a little secret: no one, including us, enjoys writing timed essays. But a little practice goes a long way. If you want to head into your AP English Exam with a cool head, you’ll want to know what you’re getting into ahead of time. We can’t promise the AP Lang Argument Essay will ever feel like an island vacation, but we do have tons of hand tips and tricks (plus a sample essay!) below to help you do your best. This article will cover: 1) What is the AP Lang Argumentative Essay? 2) AP Lang Argument Rubric 3) AP Lang Argument Sample Prompt 4) AP Lang Argument Essay Example 5) AP Lang Argument Essay Example: Answer Breakdown.

What is the AP Lang Argument Essay?

The AP Lang Argument Essay is one of three essays included in the written portion of the AP English Exam. The full AP English Exam is 3 hours and 15 minutes long, with the first 60 minutes dedicated to multiple-choice questions. Once you complete the multiple-choice section, you move on to three equally weighted essays that ask you to synthesize, analyze, and interpret texts and develop well-reasoned arguments. The three essays include:

Synthesis essay: You’ll review various pieces of evidence and then write an essay that synthesizes (aka combines and interprets) the evidence and presents a clear argument. Read our write-up on How to Write the AP Lang Synthesis Essay here.

Argumentative essay: You’ll take a stance on a specific topic and argue your case.

Rhetorical essay: You’ll read a provided passage, then analyze the author’s rhetorical choices and develop an argument that explains why the author made those rhetorical choices. Read our write-up on How to Write the AP Lang Rhetorical Essay here.

AP Lang Argument Essay Rubric

The AP Lang Argument Essay is graded on 3 rubric categories : Thesis, Evidence and Commentary, and Sophistication . How can you make sure you cover all three bases in your essay? We’ll break down each rubric category with dos and don’ts below:

  • Thesis (0-1 point)

When it comes to grading your thesis, AP Exam graders are checking off a box: you either have a clear thesis or you don’t. So, what crucial components of a thesis will get you your check mark?

  • Make sure your thesis argues something . To satisfy your graders, your thesis needs to take a clear stance on the issue at hand.
  • Include your thesis statement in your intro paragraph. The AP Lang Argumentative essay is just that: an essay that makes an argument, so make sure you present your argument right away at the end of your first paragraph.
  • A good test to see if you have a thesis that makes an argument for your AP Lang Argumentative Essay: In your head, add the phrase “I agree/disagree that…” to the beginning of your thesis. If what follows doesn’t logically flow after that phrase (aka if what follows isn’t an agreement or disagreement), it’s likely you’re not making an argument.
  • In your thesis, outline the evidence you’ll cover in your body paragraphs.

AP Lang Argument Essay Rubric (Continued)

  • Avoid a thesis that merely restates the prompt.
  • Avoid a thesis that summarizes the text but does not make an argument.
  • Avoid a thesis that weighs the pros and cons of an issue. Your job in your thesis is to pick a side and stick with it.
  • Evidence and Commentary (0-4 points)

This rubric category is graded on a scale of 0-4 where 4 is the highest grade. Unlike the rhetorical and synthesis essays, the evidence you need to write your AP Lang Argument Essay is not provided to you. Rather, you’ll need to generate your own evidence and comment upon it.

What counts as evidence?

Typically, the AP Lang Argument Essay prompt asks you to reflect on a broad cultural, moral, or social issue that is open to debate. For evidence, you won’t be asked to memorize and cite statistics or facts. Rather, you’ll want to bring in real-world examples of:

  • Historical events
  • Current-day events from the news
  • Personal anecdotes

For this essay, your graders know that you’re not able to do research to find the perfect evidence. What’s most important is that you find evidence that logically supports your argument.

What is commentary?

In this essay, it’s important to do more than just provide examples relevant evidence. After each piece of evidence you include, you’ll need to explain why it’s significant and how it connects to your main argument. The analysis you include after your evidence is commentary .

  • Take a minute to brainstorm evidence that logically supports your argument. If you have to go out of your way to find the connection, it’s better to think of different evidence.
  • Include multiple pieces of evidence. There is no magic number, but do make sure you incorporate more than a couple pieces of evidence that support your argument.
  • Make sure you include more than one example of evidence, too. Let’s say you’re working on an essay that argues that people are always stronger together than apart. You’ve already included an example from history: during the civil rights era, protestors staged group sit-ins as a powerful form of peaceful protest. That’s just one example, and it’s hard to make a credible argument with just one piece of evidence. To fix that issue, think of additional examples from history, current events, or personal experience that are not related to the civil rights era.
  • After you include each piece of evidence, explain why it’s significant and how it connects to your main argument.
  • Don’t summarize or speak generally about the topic. Everything you write must be backed up with specific and relevant evidence and examples.
  • Don’t let quotes speak for themselves. After every piece of evidence you include, make sure to explain and connect the evidence to your overarching argument.

AP Lang Argument Essay (Continued)

  • Sophistication (0-1 point)

According to the College Board , one point can be awarded to AP Lang Argument essays that achieve a high level of sophistication. You can accomplish that in four ways:

  • Crafting a nuanced argument by consistently identifying and exploring complexities or tensions.
  • Articulating the implications or limitations of an argument by situating it within a broader context.
  • Making effective rhetorical choices that consistently strengthen the force and impact of the student’s argument.
  • Employing a style that is consistently vivid and persuasive.

In sum, this means you can earn an additional point for going above and beyond in depth, complexity of thought, or by writing an especially persuasive, clear, and well-structured essay. In order to earn this point, you’ll first need to do a good job with the fundamentals: your thesis, evidence, and commentary. Then, to earn your sophistication point, follow these tips:

  • Outline your essay before you begin to ensure it flows in a clear and cohesive way.
  • Include well-rounded evidence. Don’t rely entirely on personal anecdotes, for example. Incorporate examples from current events or history, as well.
  • Thoroughly explain how each piece of evidence connects to your thesis in order to fully develop your argument.
  • Explore broader implications. If what you’re arguing is true, what does that mean to us today? Who is impacted by this issue? What real-world issues are relevant to this core issue?
  • Briefly explore the other side of the issue. Are the instances where your argument might not be true? Acknowledge the other side, then return to proving your original argument.
  • Steer clear of generalizations (avoid words like “always” and “everyone”).
  • Don’t choose an argument you can’t back up with relevant examples.
  • Avoid complex sentences and fancy vocabulary words unless you use them often. Long, clunky sentences with imprecisely used words are hard to follow.

AP Lang Argument Sample Prompt

The sample prompt below is published online by the College Board and is a real example from the 2021 AP English Exam. The prompt provides background context, essay instructions, and the text you need to analyze.

Suggested time—40 minutes.

Many people spend long hours trying to achieve perfection in their personal or professional lives. Similarly, people often demand perfection from others, creating expectations that may be challenging to live up to. In contrast, some people think perfection is not attainable or desirable.

Write an essay that argues your position on the value of striving for perfection.

In your response you should do the following:

  • Respond to the prompt with a thesis that presents a defensible position.
  • Provide evidence to support your line of reasoning.
  • Explain how the evidence supports your line of reasoning.
  • Use appropriate grammar and punctuation in communicating your argument.

AP Lang Argument Essay Example

As the old phrase says, “Practice makes perfect.” But is perfection something that is actually attainable? Sometimes, pushing for perfection helps us achieve great things, but most often, perfectionism puts too much pressure on us and prevents us from knowing when we have done the best we can. Striving for perfection can only lead us to shortchange ourselves. Instead, we should value learning, growth, and creativity and not worry whether we are first or fifth best.

Students often feel the need to be perfect in their classes, and this can cause students to struggle or stop making an effort in class. In elementary and middle school, for example, I was very nervous about public speaking. When I had to give a speech, my voice would shake, and I would turn very red. My teachers always told me “relax!” and I got Bs on Cs on my speeches. As a result, I put more pressure on myself to do well, spending extra time making my speeches perfect and rehearsing late at night at home. But this pressure only made me more nervous, and I started getting stomach aches before speaking in public.

Once I got to high school, however, I started doing YouTube make-up tutorials with a friend. We made videos just for fun, and laughed when we made mistakes or said something silly. Only then, when I wasn’t striving to be perfect, did I get more comfortable with public speaking.

AP Lang Argumentative Essay Example (Continued)

In the world of art and business and science, perfectionism can also limit what we are able to achieve. Artists, for example, have to take risks and leave room for creativity. If artists strive for perfection, then they won’t be willing to fail at new experiments and their work will be less innovative and interesting. In business and science, many products, like penicillin for example, were discovered by accident. If the scientist who discovered penicillin mold growing on his petri dishes had gotten angry at his mistake and thrown the dishes away, he would never have discovered a medicine that is vital to us today.

Some fields do need to value perfection. We wouldn’t like it, for example, if our surgeon wasn’t striving for perfection during our operation. However, for most of us, perfectionism can limit our potential for learning and growth. Instead of trying to be perfect, we should strive to learn, innovate, and do our personal best.

AP Lang Argument Essay Example: Answer Breakdown

The sample AP Lang Argumentative Essay above has some strengths and some weaknesses. Overall, we would give this essay a 3 or a 4. Let’s break down what’s working and what could be improved:

  • The essay offers a thesis that makes a clear argument that is relevant to the prompt: “Striving for perfection can only lead us to shortchange ourselves. Instead, we should value learning, growth, and creativity and not worry whether we are first or fifth best.”
  • The first body paragraph provides evidence that supports the essay’s thesis. This student’s personal anecdote offers an example of a time when perfectionism led them to shortchange themselves.
  • The second body paragraph provides additional evidence that supports the essay’s thesis. The example describing the discovery of penicillin offers another example of a situation in which perfectionism might have limited scientific progress.
  • The writer offers commentary explaining how her examples of public speaking and penicillin illustrate that we should “value learning, growth, and creativity” over perfectionism.
  • The essay follows one line of reasoning and does not stray into tangents.
  • The essay is organized well with intro, body, and concluding paragraphs. Overall, it is easy to read and is free of grammar errors.

What could be improved:

  • Although the second body paragraph provides one good specific example about the discovery of penicillin, the other examples it offers about art and business are only discussed generally and aren’t backed up with evidence. This paragraph would be stronger if it provided more examples. Or, if this writer couldn’t think of examples, they could have left out mentions of art and business altogether and included alternate evidence instead.
  • This writer would more thoroughly support their argument if they were able to offer one more example of evidence. They could provide another personal anecdote, an example from history, or an example from current events.
  • The writer briefly mentions the other side of the argument in their concluding paragraph: “Some fields do need to value perfection. We wouldn’t like it, for example, if our surgeon wasn’t striving for perfection during our operation.” Since it’s so brief a mention of the other side, it undermines the writer’s overall argument. This writer should either dedicate more time to reflecting on why even surgeons should “value learning, growth, and creativity” over perfectionism, or they should leave these sentences out.

AP Lang Argument Essay Example—More Resources

Looking for more tips to help you master your AP Lang Argumentative Essay? Brush up on 20 Rhetorical Devices High School Students Should Know and read our Tips for Improving Reading Comprehension .

If you’re ready to start studying for another part of the AP English Exam, find more expert tips in our How to Write the AP Lang Synthesis and How to Write the AP Lang Rhetorical Essay blog posts.

  • High School Success

' src=

Christina Wood

Christina Wood holds a BA in Literature & Writing from UC San Diego, an MFA in Creative Writing from Washington University in St. Louis, and is currently a Doctoral Candidate in English at the University of Georgia, where she teaches creative writing and first-year composition courses. Christina has published fiction and nonfiction in numerous publications, including The Paris Review , McSweeney’s , Granta , Virginia Quarterly Review , The Sewanee Review , Mississippi Review , and Puerto del Sol , among others. Her story “The Astronaut” won the 2018 Shirley Jackson Award for short fiction and received a “Distinguished Stories” mention in the 2019 Best American Short Stories anthology.

  • 2-Year Colleges
  • Application Strategies
  • Best Colleges by Major
  • Best Colleges by State
  • Big Picture
  • Career & Personality Assessment
  • College Essay
  • College Search/Knowledge
  • College Success
  • Costs & Financial Aid
  • Dental School Admissions
  • Extracurricular Activities
  • Graduate School Admissions
  • High Schools
  • Law School Admissions
  • Medical School Admissions
  • Navigating the Admissions Process
  • Online Learning
  • Private High School Spotlight
  • Summer Program Spotlight
  • Summer Programs
  • Test Prep Provider Spotlight

College Transitions Sidebar Block Image

“Innovative and invaluable…use this book as your college lifeline.”

— Lynn O'Shaughnessy

Nationally Recognized College Expert

College Planning in Your Inbox

Join our information-packed monthly newsletter.

I am a... Student Student Parent Counselor Educator Other First Name Last Name Email Address Zip Code Area of Interest Business Computer Science Engineering Fine/Performing Arts Humanities Mathematics STEM Pre-Med Psychology Social Studies/Sciences Submit

argumentative essay example ap lang

Crafting an Impressive Argumentative Essay for AP Lang

argumentative essay example ap lang

Writing an argumentative essay for AP Language and Composition requires a strategic approach to effectively convey your perspective. Here's a guide to crafting an impressive argumentative essay:

1. Understand the Prompt:

   - Tip: Carefully read and analyze the prompt. Identify the key elements, including the task you are asked to perform and any specific requirements or constraints.

2. Develop a Clear Thesis Statement:

   - Tip: Formulate a concise and focused thesis statement that clearly states your argument or position. This statement should guide the reader on what to expect in your essay.

3. Identify Your Audience:

   - Tip: Consider your target audience and tailor your language and arguments accordingly. Understanding your audience helps you establish a connection and effectively convey your message.

4. Organize Your Essay Effectively:

   - Tip: Structure your essay with a clear introduction, body paragraphs, and a conclusion. Ensure a logical flow of ideas, with each paragraph contributing to the overall argument.

5. Provide Context and Background:

   - Tip: Begin with a brief introduction that provides context for your argument. Clearly state the issue at hand and offer background information to help readers understand the significance of your argument.

6. Present a Strong Claim:

   - Tip: Clearly articulate your main claim or argument. This should be a debatable statement that forms the core of your essay. Avoid vague or overly broad claims.

7. Support Your Argument with Evidence:

   - Tip: Use relevant and compelling evidence to support your claims. This can include facts, statistics, examples, anecdotes, or quotations. Ensure that your evidence is credible and directly contributes to your argument.

8. Address Counterarguments:

   - Tip: Acknowledge potential counterarguments and address them in your essay. This demonstrates a nuanced understanding of the issue and strengthens your overall argument.

9. Use Persuasive Language:

    - Tip: Choose language that is persuasive and impactful. Utilize rhetorical devices, vivid imagery, and compelling language to engage your readers and convey the urgency or importance of your argument.

10. Craft Well-Reasoned Body Paragraphs:

    - Tip: Dedicate separate paragraphs to different aspects of your argument. Each paragraph should have a clear topic sentence, supporting evidence, and a concise explanation of how the evidence relates to your thesis.

11. Ensure Cohesive Transitions:

    - Tip: Use transitional phrases and sentences to ensure a smooth transition between paragraphs. This enhances the coherence of your essay and guides readers through your line of reasoning.

12. Conclude Effectively:

    - Tip: Summarize your main points in the conclusion and restate the significance of your argument. Avoid introducing new information in the conclusion. End with a strong closing statement that leaves a lasting impression.

13. Revise and Edit:

    - Tip: Set aside time for revision and editing. Review your essay for clarity, coherence, and grammatical accuracy. Ensure that your argument is compelling and well-supported.

14. Seek Feedback:

    - Tip: If possible, seek feedback from peers, teachers, or mentors. External perspectives can provide valuable insights and help you identify areas for improvement.

15. Stay Focused on the Argument:

    - Tip: Throughout the essay, stay focused on your central argument. Avoid unnecessary tangents or diverging from the main point. Every part of your essay should contribute to strengthening your overall argument.

Crafting an impressive argumentative essay for AP Language and Composition involves a combination of persuasive writing techniques, solid evidence, and effective organization. By following these tips, you can create a compelling essay that showcases your ability to engage in sophisticated argumentation and rhetorical analysis.

You Might Also Like

argumentative essay example ap lang

Guidelines To Write Impressive High School Resume

Know some important guidelines will help you write an effective high school resume that will stand out in the crowd. Check out resume sample here

argumentative essay example ap lang

The Secret to Getting off the Waitlist

So how would you go about making a letter of continued interest while you’re on the waiting status? Here’s a guide we’ve got for you.

argumentative essay example ap lang

The Perfect College Essay Structure

The Fundamentals of writing an Essay which includes the process of brainstorming, drafting, and finalizing.

AP Guru has been helping students since 2010 gain admissions to their dream universities by helping them in their college admissions and SAT and ACT Prep

Free Resources

404 Not found

404 Not found

logo-type-white

AP® English Language

How to get a 6 on the argument frq in ap® english language.

  • The Albert Team
  • Last Updated On: March 1, 2022

How to Get a 6 on the Argument FRQ in AP® English Language

What We Review

Introduction: How to Get a 6 on the Argument FRQ in AP® English Language

Wondering how to get a 6 on the argumentative essay in AP® English Language? 

To score an 5 on the AP® English Argument FRQ question, the CollegeBoard scoring guidelines outline that students need to write an essay that effectively argues a position, uses appropriate and convincing evidence, and showcases a wide range of the elements of writing. Essays that score a 6 do all of that and, additionally, demonstrate sophistication in their argument.

An essay that does all of that is an incredibly well-constructed essay. Such an essay needs a solid framework and excellent support. To do this, it is important to have a clear idea of what you are being asked, to not waffle, to spend time and care with your thesis and outline, and to support every claim you make.

We know the best way to write an AP® English FRQ that does everything right is to understand what you are going to see on the AP® English Language test. Read on to prepare yourself for exam day and earn that 6!

What to Expect from the AP® English Language Argument Free Response Questions

The AP® English argument FRQ is the most straightforward of the AP® English FRQs because it is the most similar to the essays you’re already used to writing. It’s exciting to have free reign and make your own argument, unrestrained from rhetorical analysis devices or documents. But, like most AP® writing, it also can be a little overwhelming.

There’s nothing to read and analyze to provide evidence or help you form an argument. Whether you’re feeling excited or overwhelmed by the AP® writing argument FRQ, consider the rhetorical situation. Be strategic about forming your thesis, craft a strong, chronological argument, and utilize good, supportive evidence to earn a better overall essay response.

Determine the question.

The first question to ask yourself is what am I being asked to do ? This may seem obvious, but it’s surprising how tricky it can be to figure out. Look for keywords and phrases that will answer that question.

Here’s an example from the 2019 AP® English Language argumentative essay.

What to Expect from the AP® English Language Argument Free Response Questions - Determine the Question

Though there are just two short paragraphs, there is a lot of room for confusion here. In this case, “Then, write a well-developed essay in which you explain your judgment.” is the key sentence you are looking for. In 2019, AP® English Language test takers were asked to select a concept, place, role, etc. that they believed was “overrated,” and explain why.

If you cannot determine what the question is, go back and reread the prompt. Focus on the last few sentences, as that’s where you’ll usually find it.

Knowing the question you are answering is the most important part of AP® writing. You will not be able to answer the question effectively if you aren’t certain what the question is. Pick out a specific sentence or two to determine the question, and thereby ensure that you aren’t just writing an essay that responds to the general sense of the argument essay prompts

Pick an opinion and stick to it.

The next step is both simple and difficult. Identify your own opinion on the subject.

But remember — the AP® argumentative essay exam format is designed to test how well you can craft an argument. Questions like the 2019 question seem so daunting, because claiming anything to be “overrated” is such a broad topic. It is a bigger question than students are used to encountering on an AP® test.

But, always remember, there is no right or wrong answer for this AP® English FRQ. And whatever argument you choose will not come back later in the exam or in your final grade in the class. This is not to say that you shouldn’t believe in what you are writing. Only that you should remember that both sides are arguable, pick one, and stick to it. Don’t waffle.

Below we break down two sample student answers from this same 2019 prompt. 

What to Expect from the AP® English Language Argument Free Response Questions - PIck an opinion negative example

In this AP® Lang argument essay example, the student jumps from describing places, to people, to outfits. The prompts asked for only one example and the student gives three.  By doing this, it shows they were not only unable to grasp what the prompt was asking, but that they couldn’t stick to their opinion.  Instead of deeply strengthening one choice, the student gives vague, half-reasons for too many choices. When writing your FRQs, choose just one concept and stick to it.

The following example demonstrates a strong student response:

What to Expect from the AP® English Language Argument Free Response Questions - Pick an opinion strong example

This student picks one clear concept, capitalism, and clearly outlines their support for it.  They write with clear language that opens the door for the deeper analysis coming later in the essay.

Like this student, choose just one clear argument to delve into when writing your FRQ.

Craft a thesis statement.

The thesis statement should be both simple and elegant. Students often find it one of the more difficult writing skills to master, but we’re here to help. Just remember that it should encompass your entire essay in just one sentence.  So, for the 2019 argument FRQ :

Good thesis: While capitalism undeniably has its upsides, it has many downsides that are rarely recognized. When considering the downsides, capitalism is clearly overrated as it commodifies humanity and uplifts a minority at the expense of the majority.

This thesis breaks down a) that the author clearly states his claim that capitalism is overrated, b) that the author will support that claim with examples on how it commodifies humanity and how it hurts the majority in favor of the minority.

Good thesis: While the Electoral College was created in the name of equality for smaller states, it is ultimately overrated because it undercuts the popular vote, it is an archaic practice that is unsuitable for the modern era.

This thesis claims the Electoral College is overrated by claiming it doesn’t do what it was created to do in the first place- support equality.  It also introduces two supporting examples for the rest of the essay- it undercuts the popular vote and it doesn’t work in the modern era.

Not a good thesis: Kicking a ball in a net and scoring, is not as important as saving lives. Soccer to me would be considered overrated.

This thesis doesn’t give clear direction for the rest of the essay.  The author claims soccer is overrated, but doesn’t tell us why. The example that “it’s not as important as saving lives” is unrelated and also not touched on again later in the essay. This thesis isn’t specific and doesn’t give you a clear idea of what the author will be saying next.

Not a good thesis: The term “overrated” has been used in conversation to diminish the value of roles. In unusual circumstances the term “overrated” should be applied to the idea of freedom in regards to social change, but overall it should not be applied in regards to global devastation and cruel treatment.

This thesis does not directly answer the question.  Is the author arguing that freedom is overrated? They also claim that the term overrated doesn’t apply to global devastation and cruel treatment. This second claim is both unrelated to the first and doesn’t work to answer the initial prompt.

Looking at these four examples, can you see the difference between a strong and weak thesis?

After you’ve determined your thesis, use it as a jumping point to sketch a quick outline. Then, follow your outline, bringing in your own concrete examples and evidence. Doing so will improve your AP® writing.

Return to the Table of Contents

Craft a chronological argument.

A good argument builds as you move through the essay. It does not simply repeat the same points. Instead, the different points of the argument build off one another and work together to advance the author’s point.

Let’s look at the 2018 AP® English argument FRQ for an example.

What to Expect from the AP® English Language Argument Free Response Questions - Craft a chronological argument

In this case, students are being asked to argue a position on the value of choosing the unknown. 

All students are likely to have their own definitions of what “choosing the unknown” might mean. You first want to consider what this phrase means to you, and how it applies to the real world.  Could it mean breaking out of your comfort zone in daily routines, or could it mean going to theater school to follow your dreams?  There’s no wrong answers, but try to pin down one. Consider Lindbergh’s quote the prompt gives you, and how shock, disappointment, and enrichment play into choosing the unknown.

Once you’ve nailed down your definition, you can begin to form your arguments. A chronological argument builds off itself. So, in this question’s case, an outline would look something like this:

  • Choosing the unknown is necessary for the development of the human race.
  • Scientific advancements cannot be made without testing the boundaries of the unknown.
  • Cultural and artistic growth can only occur through exploring the unknown.

First, a student must define what choosing the unknown means, and what makes it difficult. Next the student argues for the value of choosing the unknown, in that the human race could never develop without it.  Finally, the student will argue for the invaluable scientific and cultural/artistic advances made throughout history by breaking known boundaries.

When you sketch your outline, quickly ask yourself if the outline would make just as much sense if you rearranged it. If the answer is no, start writing your essay. If the answer is yes, try to structure your argument so that your points build off one another.

Support your claims.

All arguments need evidence. This is the proof you need to support your thesis. And in the case of the AP® English argument FRQ, the evidence all comes from you. What exactly that evidence is will vary from question to question and from student to student. But make sure that every point you make is supported by evidence.

Here’s some good news — you already know quite a bit about effective evidence from what you have learned in AP® English about rhetorical devices. Your main purpose in this essay is to persuade. What have you learned in class about effective ways to persuade? What rhetorical devices can you utilize? Try to pick the best devices to support your argument that you can.

Here are some examples of supportive and non-supportive evidence that students could use to support their claims.

What to Expect from the AP® English Language Argument Free Response Questions - Support your claims

The 2017 AP® English language argument FRQ asked students to argue a position if the most essential skill is artifice. The example student answers given below are from here .

Supportive evidence:   “Throughout history, rulers have utilized countless different methods of achieving power, however none have been so successful as mastering the art of lying.

In his advice to future rulers, Niccolo Machiavelli encouraged them to lie and maintain the illusion of sympathy to the common struggles in order to retain power. He asserts that it is imperative for a ruler to appear caring and sympathetic even if he has no objective but power.

Machiavelli argues that to be sincere and honest is akin to being vulnerable. A ruler must be skilled in the art of deception if he is not to fall prey to usurpers. Thus, it is essential that he appear humble and morally upright to his constituents as he is to appear idealistic, despite his nature being identical to his citizens.”

In this paragraph, the student chooses to discuss the role of artifice in politics. The student claims that mastering lying is essential to achieving political power. The student uses Machiavelli’s leadership and beliefs as specific examples to support this, by analyzing and connecting each point back to his/her claim.

Non-supportive evidence: “Another example would be actors on red carpets or at interviews they sound generous and relatable, but in reality they could be selfish people who don’t care about anyone. To the public they act charming, honest, and sincere. They do this so they can get famous and rich. They do this so they will never get ignored.”

In this paragraph, the student chooses to discuss the role of artifice in the culture of entertainment and celebrities. However, the student does not utilize supportive evidence to do so. The paragraph is full of claims about how actors lie, but does not provide a concrete example to anchor the claims. The student provides a lot of very vague generalizations, but no clear evidence or examples of specific celebrities and how they used artifice to succeed.

There is so much variance in prompts and students’ prior knowledge; it’s impossible to provide a checklist of what makes evidence supportive. But a good trick to decide if you’ve supported your claims well enough is to talk to yourself. No really, it’s a good idea.

Picture yourself discussing your essay with someone. Imagine that this person disagrees with everything that you say. Every time you make a claim, like that it’s important to be polite in an email, your imaginary person shakes their head and tells you no. How would you try to convince them? What examples would you use? Make sure that for each opinion you put forward; you have provided an answer to someone who would disagree with you.

The evidence is an important part of your essay. If your outline and your argument are a framework, your evidence is the brick and mortar. A house without brick and mortar won’t fall, but it won’t be a very nice house to inhabit. Tie every claim you make to a piece of evidence to ensure the best essay possible.

Wrapping Things Up: Scoring a 6 on the Argument FRQ for AP® English Language

The AP® English argument FRQ varies quite a bit. But it is ultimately about how well you can put forth an argument. So, don’t be afraid to spend some time crafting that argument. We’ve covered a lot in this article- here are the main points to remember:

  • Determine the question. Figure out what the prompt is asking you to do.
  • Pick an opinion and stick to it. Choose one side of the argument and one clear claim to support all the way through.
  • Craft a thesis statement. Your thesis should be clear, concise, and introduce the content of your essay.
  • Craft a chronological argument. Make an argument that builds on its prior points.
  • Support your claims. Support yourself with concrete, specific evidence and examples. 

But most of all, have fun. This essay is the one you should be looking forward to, where you have the freest rein. Enjoy it and earn yourself a 6.

Do the examples shown make sense to you? Can you picture yourself moving through the AP® writing argument FRQ with ease now?

Interested in a school license?​

8 thoughts on “how to get a 6 on the argument frq in ap® english language”.

Thank you for explaining this so eloquently. Excellent post, I will keep this handy and refer to it often from now on. It’s so educative. Great post!

Sure, glad it helped.

I’m an AP® Language teacher and the title of your article caught my eye because the essays aren’t scored on a 0-9 scale anymore. The max score for an essay now is a 6. Essays are now scored in 3 categories: Thesis: 0 or 1 point Evidence and commentary: 0-4 points Sophistication: 0 or 1 point I just wanted to let you know! I saw this was last updated in 2020 and just thought it should reflect the current AP® exam.

Thank you for the heads up! This is an older blog post that must have had something else updated to it this year. We’ve gone ahead and revised the post.

Hi, my AP® Language teacher emphasized on a counterargument at the end of the supporting paragraphs. Could you elaborate on it? Also, how exactly do we get the sophistication point?

Hi Stephanie, thanks for reaching out! Making a solid counter-argument is definitely one way to make sure that you earn the Sophistication point. We recommend having a look at our AP® English Language Review Guide for 2021 for more tips! The College Board’s Free-Response Question and Scoring Information Archive also provides authentic examples of student writing — many of which successfully make counterarguments and rebuttals to earn the Sophistication point.

Hi can I get a 6?

Hi Roy, we certainly believe that earning a 6 on your FRQs is possible with practice and dedication! I’d recommend having a look at our AP® English Language Review Guide for tips and tricks, and you can also browse our AP® English Language and Composition Resource Page and Free Response practice questions for targeted practice.

Comments are closed.

Popular Posts

AP® Physics I score calculator

AP® Score Calculators

Simulate how different MCQ and FRQ scores translate into AP® scores

argumentative essay example ap lang

AP® Review Guides

The ultimate review guides for AP® subjects to help you plan and structure your prep.

argumentative essay example ap lang

Core Subject Review Guides

Review the most important topics in Physics and Algebra 1 .

argumentative essay example ap lang

SAT® Score Calculator

See how scores on each section impacts your overall SAT® score

argumentative essay example ap lang

ACT® Score Calculator

See how scores on each section impacts your overall ACT® score

argumentative essay example ap lang

Grammar Review Hub

Comprehensive review of grammar skills

argumentative essay example ap lang

AP® Posters

Download updated posters summarizing the main topics and structure for each AP® exam.

Interested in a school license?

argumentative essay example ap lang

Bring Albert to your school and empower all teachers with the world's best question bank for: ➜ SAT® & ACT® ➜ AP® ➜ ELA, Math, Science, & Social Studies aligned to state standards ➜ State assessments Options for teachers, schools, and districts.

pep

Find what you need to study

Argument Essay: Evidence

9 min read • october 30, 2020

Stephanie Kirk

Stephanie Kirk

We aren’t sure where it started, but many teachers use REHUGO to help students find evidence on the Argument FRQ .  This acronym provides a quick check that can help you build logical evidence that supports your claim .

R - Reading - Something you have read, fiction or nonfiction, that connects the given topic.

E - Entertainment - A movie or song with dialogue or lyrics that present related ideas.

H - History - An event, document, speech, or person from history that aligns with the given topic.

U - Universal Truths - A common maxim or socially-accepted quote people tend to accept as truth.

G - Government - A national or international current event or governmental situation related to the topic.

O - Observations - Any cultural, technical, or societal trend that relates to the topic.

Suggested Guided Questions for the Argument FRQ

Now that you have a better understanding of the Argument FRQ ’s expectations and scoring, let’s visit a sample prompt and add a few guided questions that you can use to help plan your own writings.

Guided Question 1: What does the prompt say?

Why do I do this? Understanding the concept or idea presented by the prompt is vital to planning a response that thoroughly addresses the prompt and stays on topic throughout.  This is where you are going to BAT the PROMPT .

What does it look like?   Break down the prompt to identify what the given assertion is saying to decide what your position on the given topic will be.

Background : Gladwell asserts that society should trust each other in order to continue to be productive. Assuming the best about each other presents a better outcome than assuming the worst about each other.

Advice : The new stable prompt wording does not give much advice , but you should revisit advice you learned in class or from us as Fiveable -- things like using Toulmin to plan your response and planning modes of development that help progress your reasoning.

Task : Write an essay giving your position about the importance of trust. Specifically, is Gladwell right or wrong? And why?

🎥 Watch: AP Lang - Argumentation, Part I: It's a Trap!

Rhetorical Situation : When writing for AP Lang, it is important to consider the rhetorical situation and write in a manner that demonstrates an understanding of all elements of that situation. 

Context - the historical, social, and cultural movements in the time of the text

Occasion - special circumstances, atmosphere, attitudes, and events surrounding the creation of the text

Exigence - the urgency that leads to an action

Purpose - the goal the speaker wants to achieve and the desired audience movement

Speaker - the person or group who creates a text

Persona - the “mask” shown to his/her audience

Audience - listener, viewer, or reader of a text

Think : The goal is to present the information in a way the audience will understand ( diction / syntax ). May have a primary audience , but a secondary or unintended audience may be created if the text ends up on social media, etc.

https://i.ibb.co/jzPqSvS/Screen-Shot-2020-03-08-at-2-55-36-PM.png

Image Courtesy of Dana Anderson, Writing Unleashed

Guided Question 2: What do I think?

Why do I do this?   Taking a moment to brainstorm ideas can help organize thoughts and build an outline that you can revisit if you lose your train of thought in the stress of timed writing.  

What does it look like?   This might just be stream-of-consciousness in your head, cloud diagrams, or even bulleted notes on the side of your prompt, but it needs to end with a clear position statement you can use for your thesis statement . For example:

Trust is important. It does suck to get betrayed though but having a positive outlook creates positive results. Thinking the worst makes people act negatively because they project in a way that leads toward the worst response. ⬇️

Thesis Statement : Although some people believe humanity seems self-interested, a trusting nature enables individuals to focus on the positive and treat others with the respect that foster positive interrelationships.

Guided Question 3: What evidence can I use?

Why do I do this?   Revisit REHUGO and use Toulmin to plan your body paragraphs based on the thesis statement you came to when brainstorming ideas.

Modes of Development : When writing, it is helpful to arrange the overall essay and its parts in a way that aligns with the purpose .  Consider these basic modes and how you might use them in writing an argumentative essay.

Cause and Effect attempt to follow the chain of events and establish causation. The description brings imagery and details into a text so that it set up the tone and ensures the reader can follow the mood.

Classification allows the speaker to categorize things in a way that guides the reader to follow the line of reasoning.

Comparison , looking at the similarities and differences, helps to analyze the intricate details of a given topic.  Because this describes differing elements, it may be structured by the element or by the characteristic.

Definition works to explain what something is or is not. By defining the subject being discussed, the speaker is able to control the thinking about that subject.  Because this helps to clarify the topic, it is generally used in the introductory section of argumentation.

Exemplification is used when explaining the topic or situation by giving examples to help lead the audience to the desired conclusion.

Narration tells a story or gives an anecdote to help illustrate the point.

Process Analysis serves to explain the process by which something is done.

What does it look like?   I always use a version of the T-chart because one side is my evidence and the other side helps me consider multiple perspectives.  You may not have an idea in all areas, and you may have multiple ideas in one area. Try to time yourself so that you get plenty of strong evidence without wasting too much time.

🎥 Watch: AP Lang - Review: Argument Body Paragraphs

PRO TIP: What if you have more evidence for the other side? Well, you haven’t started writing just yet, so it isn’t too late to go back and revise the thesis statement . Sometimes this happens in looking for evidence, and that’s perfectly okay.  In fact, many times students will say they wrote an essay using evidence that went completely against what they felt in their head or heart just because they couldn’t put a logical argument on paper without getting too emotional. Know your audience , and build your argument.

Guided Question 4: How should I effectively organize my response?

Why do I do this?   This step helps to outline the response so that your ideas are organized before you start writing.  The general advice is to follow the structure of Classical Argumentation , but there is no rule saying that must be done to score well on the rubric. 💯

What does it look like?   One way of doing this would be to mark numbers by ideas tracked and written in the brainstorm, but some do take a moment to build an outline with topic sentences. 

Start by creating Toulmin Position Statements that can be used for topic sentences and then move into a writing plan. Here’s a sample for this prompt:

Sample Outline:

Intro - Summarize idea, define trust

Revised Thesis: Although some people believe trust must be earned, maintaining a trusting nature is important because it enables society to focus on positivity and create positive interrelationships that lead to positive outcomes. 

At the most basic level, trust is important because the human experience has taught us that the foundation of a relationship is trust.

Universal truth

Observation

Indeed historical perspective can give more credible evidence as to why trust is vital to society.

Certainly, there have been times where trust resulted in less-than-desirable situations.

Entertainment - refute

Conclude: When considering the value of trust in society, it is clear that the benefits of granting trust far outweigh the consequences of withholding it.

Key Terms to Review ( 35 )

Argument FRQ

BAT the PROMPT

Cause and Effect

Classical Argumentation

Classification

Description

Entertainment

Exemplification

Logical Evidence

Modes of Development

Observations

Process Analysis

Rhetorical Situation

Thesis Statement

Toulmin method

Trust in modern society

Universal Truths

Fiveable

Stay Connected

© 2024 Fiveable Inc. All rights reserved.

AP® and SAT® are trademarks registered by the College Board, which is not affiliated with, and does not endorse this website.

Calculate for all schools

Your chance of acceptance, your chancing factors, extracurriculars, what's the best approach for the ap lang argument essay.

Hey everyone! I was wondering how you guys prepare for and structure your argument essay in AP Lang. What types of evidence do you usually use? And any tips on choosing effective examples to support your argument?

Hi there! The AP Lang argument essay is all about making a strong, well-structured argument in a clear and concise manner. Here's a step-by-step approach you can follow and some tips to help you prepare:

1. Understand the prompt: Carefully read the given prompt and make sure you understand what it's asking you to do. Keep an eye out for key terms that might require closer consideration.

2. Plan your essay: After fully understanding the prompt, take a few minutes to plan your essay. Decide on your main argument, and think about how you can best support it with evidence.

3. Choose your evidence: For the AP Lang argument essay, it's important to use a mix of both evidence from the provided sources and your own knowledge. Consider using historical events, real-life examples, and even personal experiences if they're relevant and can effectively support your argument.

4. Create a clear thesis statement: This should be a concise statement that outlines your main argument and how you plan to support it. Make sure your thesis is specific and directly addresses the prompt.

5. Outline your essay: Break down your essay into an introduction, body paragraphs, and a conclusion. Plan which evidence you'll use in each body paragraph, and be sure to vary the types of evidence to make your argument more compelling.

6. Write the introduction: Begin with a hook or an interesting fact that grabs the reader's attention. Then, provide some context for your argument before clearly stating your thesis.

7. Craft clear topic sentences: Each body paragraph should start with a topic sentence that introduces the main point of that paragraph and connects it back to your thesis.

8. Use appropriate transitions: Ensure there's a smooth flow between your paragraphs and ideas by using appropriate transition words and phrases.

9. Address counterarguments: To make your argument more convincing, consider acknowledging valid counterarguments and then refuting them with evidence.

10. Write a strong conclusion: Sum up your main points, restate your thesis, and leave the reader with something thought-provoking or impactful.

To prepare for the AP Lang argument essay, practice your writing skills regularly, read a variety of sources to broaden your knowledge, and analyze sample essays to understand what works and what doesn't. Remember, the key to an effective argument essay is having a clear, focused thesis and supporting it with strong evidence. Happy writing!

About CollegeVine’s Expert FAQ

CollegeVine’s Q&A seeks to offer informed perspectives on commonly asked admissions questions. Every answer is refined and validated by our team of admissions experts to ensure it resonates with trusted knowledge in the field.

404 Not found

What are your chances of acceptance?

Calculate for all schools, your chance of acceptance.

Duke University

Your chancing factors

Extracurriculars.

argumentative essay example ap lang

How to Write the AP Lang Synthesis Essay + Example

Do you know how to improve your profile for college applications.

See how your profile ranks among thousands of other students using CollegeVine. Calculate your chances at your dream schools and learn what areas you need to improve right now — it only takes 3 minutes and it's 100% free.

Show me what areas I need to improve

What’s Covered:

What is the ap lang synthesis essay, how will ap scores affect my college chances.

AP English Language and Composition, commonly known as AP Lang, is one of the most engaging and popular AP classes offered at most high schools, with over 535,000 students taking the class . AP Lang tests your ability to analyze written pieces, synthesize information, write rhetorical essays, and create cohesive and concrete arguments. However, the class is rather challenging as only 62% of students were able to score a three or higher on the exam. 

The AP Lang exam has two sections. The first consists of 45 multiple choice questions which need to be completed in an hour. This portion counts for around 45% of your total score. These questions ask students to analyze written pieces and answer questions related to each respective passage.  All possible answer choices can be found within the text, and no prior knowledge of literature is needed to understand the passages.

The second section contains three free-response questions to be finished in under two hours and 15 minutes. This section counts for 55% of your score and includes the synthesis essay, the rhetorical essay, and the argumentative essay.

  • The synthesis essay requires you to read 6-7 sources and create an argument using at least three sources.
  • The rhetorical analysis essay requires you to describe how a piece of writing evokes specific meanings and symbolism.
  • The argumentative essay requires you to pick a perspective of a debate and create an argument based on the evidence provided.

In this post, we will take a look at the AP Lang synthesis essay and discuss tips and tricks to master this part of the exam. We will also provide an example of a well-written essay for review.  

The AP Lang synthesis essay is the first of three essays included in the Free Response section of the AP Lang exam. The exam presents 6-7 sources that are organized around a specific topic, with two of those sources purely visual, including a single quantitative source (like a graph or pie chart). The remaining 4-5 sources are text-based, containing around 500 words each. It’s recommended that students spend an hour on this essay—15 minute reading period, 40 minutes writing, and 5 minutes of spare time to check over work.

Each synthesis essay has a topic that all the sources will relate to. A prompt will explaining the topic and provide some background, although the topics are usually broad so you will probably know something related to the issue. It will also present a claim that students will respond to in an essay format using information from at least three of the provided sources. You will need to take a stance, either agreeing or disagreeing with the position provided in the claim. 

According to the CollegeBoard, they are looking for essays that “combine different perspectives from sources to form a support of a coherent position.” This means that you must state your claim on the topic and highlight relationships between several sources that support your specific position on the topic. Additionally, you’ll need to cite clear evidence from your sources to prove your point.

The synthesis essay counts for six points on the AP Lang exam. Students can receive 0-1 points for writing a thesis statement, 0-4 based on the incorporation of evidence and commentary, and 0-1 points based on the sophistication of thought and demonstration of complex understanding.

While this essay seems extremely overwhelming, considering there are a total of three free-response essays to complete, with proper time management and practiced skills, this essay is manageable and straightforward. In order to enhance the time management aspect of the test to the best of your ability, it is essential to divide the essay up into five key steps.

Step 1: Analyze the Prompt

As soon as the clock starts, carefully read and analyze what the prompt asks from you. It might be helpful to markup the text to identify the most critical details. You should only spend around 2 minutes reading the prompt so you have enough time to read all the sources and figure out your argument. Don’t feel like you need to immediately pick your stance on the claim right after reading the prompt. You should read the sources before you commit to your argument.

Step 2: Read the Sources Carefully

Although you are only required to use 3 of the 6-7 sources provides, make sure you read ALL of the sources. This will allow you to better understand the topic and make the most educated decision of which sources to use in your essay. Since there are a lot of sources to get through, you will need to read quickly and carefully.

Annotating will be your best friend during the reading period. Highlight and mark important concepts or lines from each passage that would be helpful in your essay. Your argument will probably begin forming in your head as you go through the passages, so you will save yourself a lot of time later on if you take a few seconds to write down notes in the margins. After you’ve finished reading a source, reflect on whether the source defends, challenges, or qualifies your argument.

You will have around 13 minutes to read through all the sources, but it’s very possible you will finish earlier if you are a fast reader. Take the leftover time to start developing your thesis and organizing your thoughts into an outline so you have more time to write. 

Step 3: Write a Strong Thesis Statement 

In order to write a good thesis statement, all you have to do is decide your stance on the claim provided in the prompt and give an overview of your evidence. You essentially have three choices on how to frame your thesis statement: You can defend, challenge or qualify a claim that’s been provided by the prompt. 

  • If you are defending the claim, your job will be to prove that the claim is correct .
  • If you are challenging the claim, your job will be to prove that the claim is incorrect .
  • If you choose to qualify the claim, your job will be to agree to a part of the claim and disagree with another part of the claim. 

A strong thesis statement will clearly state your stance without summarizing the issue or regurgitating the claim. The CollegeBoard is looking for a thesis statement that “states a defensible position and establishes a line of reasoning on the issue provided in the prompt.”

Step 4: Create a Minimal Essay Outline

Developing an outline might seem like a waste of time when you are up against the clock, but believe us, taking 5-10 minutes to outline your essay will be much more useful in the long run than jumping right into the essay.

Your outline should include your thesis statement and three main pieces of evidence that will constitute each body paragraph. Under each piece of evidence should be 2-3 details from the sources that you will use to back up your claim and some commentary on how that evidence proves your thesis.

Step 5: Write your Essay

Use the remaining 30-35 minutes to write your essay. This should be relatively easy if you took the time to mark up the sources and have a detailed outline.  Remember to add special consideration and emphasis to the commentary sections of the supporting arguments outlined in your thesis. These sentences are critical to the overall flow of the essay and where you will be explaining how the evidence supports or undermines the claim in the prompt.

Also, when referencing your sources, write the in-text citations as follows: “Source 1,” “Source 2,” “Source 3,” etc. Make sure to pay attention to which source is which in order to not incorrectly cite your sources. In-text citations will impact your score on the essay and are an integral part of the process.

After you finish writing, read through your essay for any grammatical errors or mistakes before you move onto the next essay.

Here are six must-have tips and tricks to get a good score on the synthesis essay:

  • Cite at least four sources , even though the minimum requirement is three. Remember not to plagiarize and cite everything you use in your arguments.
  • Make sure to develop a solid and clear thesis . Develop a stable stance for the claim and stick with it throughout the entire paper.
  • Don’t summarize the sources. The summary of the sources does not count as an argument. 
  • You don’t necessarily have to agree with the sources in order to cite them. Using a source to support a counterargument is still a good use of a source.
  • Cite the sources that you understand entirely . If you don’t, it could come back to bite you in the end. 
  • Use small quotes , do not quote entire paragraphs. Make sure the quote does not disrupt the flow or grammar of the sentence you write. 

argumentative essay example ap lang

Discover your chances at hundreds of schools

Our free chancing engine takes into account your history, background, test scores, and extracurricular activities to show you your real chances of admission—and how to improve them.

Here is an example prompt and essay from 2019 that received 5 of the 6 total points available:

In response to our society’s increasing demand for energy, large-scale wind power has drawn attention from governments and consumers as a potential alternative to traditional materials that fuel our power grids, such as coal, oil, natural gas, water, or even newer sources such as nuclear or solar power. Yet the establishment of large-scale, commercial-grade wind farms is often the subject of controversy for a variety of reasons.

Carefully read the six sources, found on the AP English Language and Composition 2019 Exam (Question 1), including the introductory information for each source. Write an essay that synthesizes material from at least three of the sources and develops your position on the most important factors that an individual or agency should consider when deciding whether to establish a wind farm.

Source A (photo)

Source B (Layton)

Source C (Seltenrich)

Source D (Brown)

Source E (Rule)

Source F (Molla)

In your response you should do the following:

  • Respond to the prompt with a thesis presents a defensible position.
  • Select and use evidence from at least 3 of the provided sources to support your line of reasoning. Indicate clearly the sources used through direct quotation, paraphrase, or summary. Sources may be cited as Source A, Source B, etc., or by using the description in parentheses.
  • Explain how the evidence supports your line of reasoning.
  • Use appropriate grammar and punctuation in communicating your argument.

[1] The situation has been known for years, and still very little is being done: alternative power is the only way to reliably power the changing world. The draw of power coming from industry and private life is overwhelming current sources of non-renewable power, and with dwindling supplies of fossil fuels, it is merely a matter of time before coal and gas fuel plants are no longer in operation. So one viable alternative is wind power. But as with all things, there are pros and cons. The main factors for power companies to consider when building wind farms are environmental boon, aesthetic, and economic factors.

[2] The environmental benefits of using wind power are well-known and proven. Wind power is, as qualified by Source B, undeniably clean and renewable. From their production requiring very little in the way of dangerous materials to their lack of fuel, besides that which occurs naturally, wind power is by far one of the least environmentally impactful sources of power available. In addition, wind power by way of gearbox and advanced blade materials, has the highest percentage of energy retention. According to Source F, wind power retains 1,164% of the energy put into the system – meaning that it increases the energy converted from fuel (wind) to electricity 10 times! No other method of electricity production is even half that efficient. The efficiency and clean nature of wind power are important to consider, especially because they contribute back to power companies economically.

[3] Economically, wind power is both a boon and a bone to electric companies and other users. For consumers, wind power is very cheap, leading to lower bills than from any other source. Consumers also get an indirect reimbursement by way of taxes (Source D). In one Texan town, McCamey, tax revenue increased 30% from a wind farm being erected in the town. This helps to finance improvements to the town. But, there is no doubt that wind power is also hurting the power companies. Although, as renewable power goes, wind is incredibly cheap, it is still significantly more expensive than fossil fuels. So, while it is helping to cut down on emissions, it costs electric companies more than traditional fossil fuel plants. While the general economic trend is positive, there are some setbacks which must be overcome before wind power can take over as truly more effective than fossil fuels.

[4] Aesthetics may be the greatest setback for power companies. Although there may be significant economic and environmental benefit to wind power, people will always fight to preserve pure, unspoiled land. Unfortunately, not much can be done to improve the visual aesthetics of the turbines. White paint is the most common choice because it “[is] associated with cleanliness.” (Source E). But, this can make it stand out like a sore thumb, and make the gargantuan machines seem more out of place. The site can also not be altered because it affects generating capacity. Sound is almost worse of a concern because it interrupts personal productivity by interrupting people’s sleep patterns. One thing for power companies to consider is working with turbine manufacturing to make the machines less aesthetically impactful, so as to garner greater public support.

[5] As with most things, wind power has no easy answer. It is the responsibility of the companies building them to weigh the benefits and the consequences. But, by balancing economics, efficiency, and aesthetics, power companies can create a solution which balances human impact with environmental preservation.

More examples can be found here at College Board.

While AP Scores help to boost your weighted GPA, or give you the option to get college credit, AP Scores don’t have a strong effect on your admissions chances . However, colleges can still see your self-reported scores, so you might not want to automatically send scores to colleges if they are lower than a 3. That being said, admissions officers care far more about your grade in an AP class than your score on the exam.

Related CollegeVine Blog Posts

argumentative essay example ap lang

PrepScholar

Choose Your Test

Sat / act prep online guides and tips, how to write a perfect synthesis essay for the ap language exam.

author image

Advanced Placement (AP)

body-pencil-sharpen-notebook-1

If you're planning to take the AP Language (or AP Lang) exam , you might already know that 55% of your overall exam score will be based on three essays. The first of the three essays you'll have to write on the AP Language exam is called the "synthesis essay." If you want to earn full points on this portion of the AP Lang Exam, you need to know what a synthesis essay is and what skills are assessed by the AP Lang synthesis essay.

In this article, we'll explain the different aspects of the AP Lang synthesis essay, including what skills you need to demonstrate in your synthesis essay response in order to achieve a good score. We'll also give you a full breakdown of a real AP Lang Synthesis Essay prompt, provide an analysis of an AP Lang synthesis essay example, and give you four tips for how to write a synthesis essay.

Let's get started by taking a closer look at how the AP Lang synthesis essay works!

Synthesis Essay AP Lang: What It Is and How It Works

The AP Lang synthesis essay is the first of three essays included in the Free Response section of the AP Lang exam.

The AP Lang synthesis essay portion of the Free Response section lasts for one hour total . This hour consists of a recommended 15 minute reading period and a 40 minute writing period. Keep in mind that these time allotments are merely recommendations, and that exam takers can parse out the allotted 60 minutes to complete the synthesis essay however they choose.

Now, here's what the structure of the AP Lang synthesis essay looks like. The exam presents six to seven sources that are organized around a specific topic (like alternative energy or eminent domain, which are both past synthesis exam topics).

Of these six to seven sources, at least two are visual , including at least one quantitative source (like a graph or pie chart, for example). The remaining four to five sources are print text-based, and each one contains approximately 500 words.

In addition to six to seven sources, the AP Lang exam provides a written prompt that consists of three paragraphs. The prompt will briefly explain the essay topic, then present a claim that students will respond to in an essay that synthesizes material from at least three of the sources provided.

Here's an example prompt provided by the College Board:

Directions : The following prompt is based on the accompanying six sources.

This question requires you to integrate a variety of sources into a coherent, well-written essay. Refer to the sources to support your position; avoid mere paraphrase or summary. Your argument should be central; the sources should support this argument .

Remember to attribute both direct and indirect citations.

Introduction

Television has been influential in United States presidential elections since the 1960's. But just what is this influence, and how has it affected who is elected? Has it made elections fairer and more accessible, or has it moved candidates from pursuing issues to pursuing image?

Read the following sources (including any introductory information) carefully. Then, in an essay that synthesizes at least three of the sources for support, take a position that defends, challenges, or qualifies the claim that television has had a positive impact on presidential elections.

Refer to the sources as Source A, Source B, etc.; titles are included for your convenience.

Source A (Campbell) Source B (Hart and Triece) Source C (Menand) Source D (Chart) Source E (Ranney) Source F (Koppel)

Like we mentioned earlier, this prompt gives you a topic — which it briefly explains — then asks you to take a position. In this case, you'll have to choose a stance on whether television has positively or negatively affected U.S. elections. You're also given six sources to evaluate and use in your response. Now that you have everything you need, now your job is to write an amazing synthesis essay.

But what does "synthesize" mean, exactly? According to the CollegeBoard, when an essay prompt asks you to synthesize, it means that you should "combine different perspectives from sources to form a support of a coherent position" in writing. In other words, a synthesis essay asks you to state your claim on a topic, then highlight the relationships between several sources that support your claim on that topic. Additionally, you'll need to cite specific evidence from your sources to prove your point.

The synthesis essay counts for six of the total points on the AP Lang exam . Students can receive 0-1 points for writing a thesis statement in the essay, 0-4 based on incorporation of evidence and commentary, and 0-1 points based on sophistication of thought and demonstrated complex understanding of the topic.

You'll be evaluated based on how effectively you do the following in your AP Lang synthesis essay:

Write a thesis that responds to the exam prompt with a defensible position

Provide specific evidence that to support all claims in your line of reasoning from at least three of the sources provided, and clearly and consistently explain how the evidence you include supports your line of reasoning

Demonstrate sophistication of thought by either crafting a thoughtful argument, situating the argument in a broader context, explaining the limitations of an argument

Make rhetorical choices that strengthen your argument and/or employ a vivid and persuasive style throughout your essay.

If your synthesis essay meets the criteria above, then there's a good chance you'll score well on this portion of the AP Lang exam!

If you're looking for even more information on scoring, the College Board has posted the AP Lang Free Response grading rubric on its website. ( You can find it here. ) We recommend taking a close look at it since it includes additional details about the synthesis essay scoring.

body-chisel-break-apart

Don't be intimidated...we're going to teach you how to break down even the hardest AP synthesis essay prompt.

Full Breakdown of a Real AP Lang Synthesis Essay Prompt

In this section, we'll teach you how to analyze and respond to a synthesis essay prompt in five easy steps, including suggested time frames for each step of the process.

Step 1: Analyze the Prompt

The very first thing to do when the clock starts running is read and analyze the prompt. To demonstrate how to do this, we'll look at the sample AP Lang synthesis essay prompt below. This prompt comes straight from the 2018 AP Lang exam:

Eminent domain is the power governments have to acquire property from private owners for public use. The rationale behind eminent domain is that governments have greater legal authority over lands within their dominion than do private owners. Eminent domain has been instituted in one way or another throughout the world for hundreds of years.

Carefully read the following six sources, including the introductory information for each source. Then synthesize material from at least three of the sources and incorporate it into a coherent, well-developed essay that defends, challenges, or qualifies the notion that eminent domain is productive and beneficial.

Your argument should be the focus of your essay. Use the sources to develop your argument and explain the reasoning for it. Avoid merely summarizing the sources. Indicate clearly which sources you are drawing from, whether through direct quotation, paraphrase, or summary. You may cite the sources as Source A, Source B, etc., or by using the descriptions in parentheses.

On first read, you might be nervous about how to answer this prompt...especially if you don't know what eminent domain is! But if you break the prompt down into chunks, you'll be able to figure out what the prompt is asking you to do in no time flat.

To get a full understanding of what this prompt wants you to do, you need to identify the most important details in this prompt, paragraph by paragraph. Here's what each paragraph is asking you to do:

  • Paragraph 1: The prompt presents and briefly explains the topic that you'll be writing your synthesis essay about. That topic is the concept of eminent domain.
  • Paragraph 2: The prompt presents a specific claim about the concept of eminent domain in this paragraph: Eminent domain is productive and beneficial. This paragraph instructs you to decide whether you want to defend, challenge, or qualify that claim in your synthesis essay , and use material from at least three of the sources provided in order to do so.
  • Paragraph 3: In the last paragraph of the prompt, the exam gives you clear instructions about how to approach writing your synthesis essay . First, make your argument the focus of the essay. Second, use material from at least three of the sources to develop and explain your argument. Third, provide commentary on the material you include, and provide proper citations when you incorporate quotations, paraphrases, or summaries from the sources provided.

So basically, you'll have to agree with, disagree with, or qualify the claim stated in the prompt, then use at least three sources substantiate your answer. Since you probably don't know much about eminent domain, you'll probably decide on your position after you read the provided sources.

To make good use of your time on the exam, you should spend around 2 minutes reading the prompt and making note of what it's asking you to do. That will leave you plenty of time to read the sources provided, which is the next step to writing a synthesis essay.

Step 2: Read the Sources Carefully

After you closely read the prompt and make note of the most important details, you need to read all of the sources provided. It's tempting to skip one or two sources to save time--but we recommend you don't do this. That's because you'll need a thorough understanding of the topic before you can accurately address the prompt!

For the sample exam prompt included above, there are six sources provided. We're not going to include all of the sources in this article, but you can view the six sources from this question on the 2018 AP Lang exam here . The sources include five print-text sources and one visual source, which is a cartoon.

As you read the sources, it's important to read quickly and carefully. Don't rush! Keep your pencil in hand to quickly mark important passages that you might want to use as evidence in your synthesis. While you're reading the sources and marking passages, you want to think about how the information you're reading influences your stance on the issue (in this case, eminent domain).

When you finish reading, take a few seconds to summarize, in a phrase or sentence, whether the source defends, challenges, or qualifies whether eminent domain is beneficial (which is the claim in the prompt) . Though it might not feel like you have time for this, it's important to give yourself these notes about each source so you know how you can use each one as evidence in your essay.

Here's what we mean: say you want to challenge the idea that eminent domain is useful. If you've jotted down notes about each source and what it's saying, it will be easier for you to pull the relevant information into your outline and your essay.

So how much time should you spend reading the provided sources? The AP Lang exam recommends taking 15 minutes to read the sources . If you spend around two of those minutes reading and breaking down the essay prompt, it makes sense to spend the remaining 13 minutes reading and annotating the sources.

If you finish reading and annotating early, you can always move on to drafting your synthesis essay. But make sure you're taking your time and reading carefully! It's better to use a little extra time reading and understanding the sources now so that you don't have to go back and re-read the sources later.

body-weightlifting-lift-strong

A strong thesis will do a lot of heavy lifting in your essay. (See what we did there?)

Step 3: Write a Strong Thesis Statement

After you've analyzed the prompt and thoroughly read the sources, the next thing you need to do in order to write a good synthesis essay is write a strong thesis statement .

The great news about writing a thesis statement for this synthesis essay is that you have all the tools you need to do it at your fingertips. All you have to do in order to write your thesis statement is decide what your stance is in relationship to the topic provided.

In the example prompt provided earlier, you're essentially given three choices for how to frame your thesis statement: you can either defend, challenge, or qualify a claim that's been provided by the prompt, that eminent domain is productive and beneficial . Here's what that means for each option:

If you choose to defend the claim, your job will be to prove that the claim is correct . In this case, you'll have to show that eminent domain is a good thing.

If you choose to challenge the claim, you'll argue that the claim is incorrect. In other words, you'll argue that eminent domain isn't productive or beneficial.

If you choose to qualify, that means you'll agree with part of the claim, but disagree with another part of the claim. For instance, you may argue that eminent domain can be a productive tool for governments, but it's not beneficial for property owners. Or maybe you argue that eminent domain is useful in certain circumstances, but not in others.

When you decide whether you want your synthesis essay to defend, challenge, or qualify that claim, you need to convey that stance clearly in your thesis statement. You want to avoid simply restating the claim provided in the prompt, summarizing the issue without making a coherent claim, or writing a thesis that doesn't respond to the prompt.

Here's an example of a thesis statement that received full points on the eminent domain synthesis essay:

Although eminent domain can be misused to benefit private interests at the expense of citizens, it is a vital tool of any government that intends to have any influence on the land it governs beyond that of written law.

This thesis statement received full points because it states a defensible position and establishes a line of reasoning on the issue of eminent domain. It states the author's position (that some parts of eminent domain are good, but others are bad), then goes on to explain why the author thinks that (it's good because it allows the government to do its job, but it's bad because the government can misuse its power.)

Because this example thesis statement states a defensible position and establishes a line of reasoning, it can be elaborated upon in the body of the essay through sub-claims, supporting evidence, and commentary. And a solid argument is key to getting a six on your synthesis essay for AP Lang!

Looking for help studying for your AP exam?

Our one-on-one online AP tutoring services can help you prepare for your AP exams. Get matched with a top tutor who got a high score on the exam you're studying for!

Get a 5 On Your AP Exam

Step 4: Create a Bare-Bones Essay Outline

Once you've got your thesis statement drafted, you have the foundation you need to develop a bare bones outline for your synthesis essay. Developing an outline might seem like it's a waste of your precious time, but if you develop your outline well, it will actually save you time when you start writing your essay.

With that in mind, we recommend spending 5 to 10 minutes outlining your synthesis essay . If you use a bare-bones outline like the one below, labeling each piece of content that you need to include in your essay draft, you should be able to develop out the most important pieces of the synthesis before you even draft the actual essay.

To help you see how this can work on test day, we've created a sample outline for you. You can even memorize this outline to help you out on test day! In the outline below, you'll find places to fill in a thesis statement, body paragraph topic sentences, evidence from the sources provided, and commentary :

  • Present the context surrounding the essay topic in a couple of sentences (this is a good place to use what you learned about the major opinions or controversies about the topic from reading your sources).
  • Write a straightforward, clear, and concise thesis statement that presents your stance on the topic
  • Topic sentence presenting first supporting point or claim
  • Evidence #1
  • Commentary on Evidence #1
  • Evidence #2 (if needed)
  • Commentary on Evidence #2 (if needed)
  • Topic sentence presenting second supporting point or claim
  • Topic sentence presenting three supporting point or claim
  • Sums up the main line of reasoning that you developed and defended throughout the essay
  • Reiterates the thesis statement

Taking the time to develop these crucial pieces of the synthesis in a bare-bones outline will give you a map for your final essay. Once you have a map, writing the essay will be much easier.

Step 5: Draft Your Essay Response

The great thing about taking a few minutes to develop an outline is that you can develop it out into your essay draft. After you take about 5 to 10 minutes to outline your synthesis essay, you can use the remaining 30 to 35 minutes to draft your essay and review it.

Since you'll outline your essay before you start drafting, writing the essay should be pretty straightforward. You'll already know how many paragraphs you're going to write, what the topic of each paragraph will be, and what quotations, paraphrases, or summaries you're going to include in each paragraph from the sources provided. You'll just have to fill in one of the most important parts of your synthesis—your commentary.

Commentaries are your explanation of why your evidence supports the argument you've outlined in your thesis. Your commentary is where you actually make your argument, which is why it's such a critical part of your synthesis essay.

When thinking about what to say in your commentary, remember one thing the AP Lang synthesis essay prompt specifies: don't just summarize the sources. Instead, as you provide commentary on the evidence you incorporate, you need to explain how that evidence supports or undermines your thesis statement . You should include commentary that offers a thoughtful or novel perspective on the evidence from your sources to develop your argument.

One very important thing to remember as you draft out your essay is to cite your sources. The AP Lang exam synthesis essay prompt indicates that you can use generic labels for the sources provided (e.g. "Source 1," "Source 2," "Source 3," etc.). The exam prompt will indicate which label corresponds with which source, so you'll need to make sure you pay attention and cite sources accurately. You can cite your sources in the sentence where you introduce a quote, summary, or paraphrase, or you can use a parenthetical citation. Citing your sources affects your score on the synthesis essay, so remembering to do this is important.

body-green-arrow-down

Keep reading for a real-life example of a great AP synthesis essay response!

Real-Life AP Synthesis Essay Example and Analysis

If you're still wondering how to write a synthesis essay, examples of real essays from past AP Lang exams can make things clearer. These real-life student AP synthesis essay responses can be great for helping you understand how to write a synthesis essay that will knock the graders' socks off .

While there are multiple essay examples online, we've chosen one to take a closer look at. We're going to give you a brief analysis of one of these example student synthesis essays from the 2019 AP Lang Exam below!

Example Synthesis Essay AP Lang Response

To get started, let's look at the official prompt for the 2019 synthesis essay:

In response to our society's increasing demand for energy, large-scale wind power has drawn attention from governments and consumers as a potential alternative to traditional materials that fuel our power grids, such as coal, oil, natural gas, water, or even newer sources such as nuclear or solar power. Yet the establishment of large-scale, commercial-grade wind farms is often the subject of controversy for a variety of reasons.

Carefully read the six sources, found on the AP English Language and Composition 2019 Exam (Question 1), including the introductory information for each source. Write an essay that synthesizes material from at least three of the sources and develops your position on the most important factors that an individual or agency should consider when deciding whether to establish a wind farm.

Source A (photo) Source B (Layton) Source C (Seltenrich) Source D (Brown) Source E (Rule) Source F (Molla)

In your response you should do the following:

  • Respond to the prompt with a thesis presents a defensible position.
  • Select and use evidence from at least 3 of the provided sources to support your line of reasoning. Indicate clearly the sources used through direct quotation, paraphrase, or summary. Sources may be cited as Source A, Source B, etc., or by using the description in parentheses.
  • Explain how the evidence supports your line of reasoning.
  • Use appropriate grammar and punctuation in communicating your argument.

Now that you know exactly what the prompt asked students to do on the 2019 AP Lang synthesis essay, here's an AP Lang synthesis essay example, written by a real student on the AP Lang exam in 2019:

[1] The situation has been known for years, and still very little is being done: alternative power is the only way to reliably power the changing world. The draw of power coming from industry and private life is overwhelming current sources of non-renewable power, and with dwindling supplies of fossil fuels, it is merely a matter of time before coal and gas fuel plants are no longer in operation. So one viable alternative is wind power. But as with all things, there are pros and cons. The main factors for power companies to consider when building wind farms are environmental boon, aesthetic, and economic factors.

[2] The environmental benefits of using wind power are well-known and proven. Wind power is, as qualified by Source B, undeniably clean and renewable. From their production requiring very little in the way of dangerous materials to their lack of fuel, besides that which occurs naturally, wind power is by far one of the least environmentally impactful sources of power available. In addition, wind power by way of gearbox and advanced blade materials, has the highest percentage of energy retention. According to Source F, wind power retains 1,164% of the energy put into the system – meaning that it increases the energy converted from fuel (wind) to electricity 10 times! No other method of electricity production is even half that efficient. The efficiency and clean nature of wind power are important to consider, especially because they contribute back to power companies economically.

[3] Economically, wind power is both a boon and a bone to electric companies and other users. For consumers, wind power is very cheap, leading to lower bills than from any other source. Consumers also get an indirect reimbursement by way of taxes (Source D). In one Texan town, McCamey, tax revenue increased 30% from a wind farm being erected in the town. This helps to finance improvements to the town. But, there is no doubt that wind power is also hurting the power companies. Although, as renewable power goes, wind is incredibly cheap, it is still significantly more expensive than fossil fuels. So, while it is helping to cut down on emissions, it costs electric companies more than traditional fossil fuel plants. While the general economic trend is positive, there are some setbacks which must be overcome before wind power can take over as truly more effective than fossil fuels.

[4] Aesthetics may be the greatest setback for power companies. Although there may be significant economic and environmental benefit to wind power, people will always fight to preserve pure, unspoiled land. Unfortunately, not much can be done to improve the visual aesthetics of the turbines. White paint is the most common choice because it "[is] associated with cleanliness." (Source E). But, this can make it stand out like a sore thumb, and make the gargantuan machines seem more out of place. The site can also not be altered because it affects generating capacity. Sound is almost worse of a concern because it interrupts personal productivity by interrupting people's sleep patterns. One thing for power companies to consider is working with turbine manufacturing to make the machines less aesthetically impactful, so as to garner greater public support.

[5] As with most things, wind power has no easy answer. It is the responsibility of the companies building them to weigh the benefits and the consequences. But, by balancing economics, efficiency, and aesthetics, power companies can create a solution which balances human impact with environmental preservation.

And that's an entire AP Lang synthesis essay example, written in response to a real AP Lang exam prompt! It's important to remember AP Lang exam synthesis essay prompts are always similarly structured and worded, and students often respond in around the same number of paragraphs as what you see in the example essay response above.

Next, let's analyze this example essay and talk about what it does effectively, where it could be improved upon, and what score past exam scorers awarded it.

To get started on an analysis of the sample synthesis essay, let's look at the scoring commentary provided by the College Board:

  • For development of thesis, the essay received 1 out of 1 possible points
  • For evidence and commentary, the essay received 4 out of 4 possible points
  • For sophistication of thought, the essay received 0 out of 1 possible points.

This means that the final score for this example essay was a 5 out of 6 possible points . Let's look more closely at the content of the example essay to figure out why it received this score breakdown.

Thesis Development

The thesis statement is one of the three main categories that is taken into consideration when you're awarded points on this portion of the exam. This sample essay received 1 out of 1 total points.

Now, here's why: the thesis statement clearly and concisely conveys a position on the topic presented in the prompt--alternative energy and wind power--and defines the most important factors that power companies should consider when deciding whether to establish a wind farm.

Evidence and Commentary

The second key category taken into consideration when synthesis exams are evaluated is incorporation of evidence and commentary. This sample received 4 out of 4 possible points for this portion of the synthesis essay. At bare minimum, this sample essay meets the requirement mentioned in the prompt that the writer incorporate evidence from at least three of the sources provided.

On top of that, the writer does a good job of connecting the incorporated evidence back to the claim made in the thesis statement through effective commentary. The commentary in this sample essay is effective because it goes beyond just summarizing what the provided sources say. Instead, it explains and analyzes the evidence presented in the selected sources and connects them back to supporting points the writer makes in each body paragraph.

Finally, the writer of the essay also received points for evidence and commentary because the writer developed and supported a consistent line of reasoning throughout the essay . This line of reasoning is summed up in the fourth paragraph in the following sentence: "One thing for power companies to consider is working with turbine manufacturing to make the machines less aesthetically impactful, so as to garner greater public support."

Because the writer did a good job consistently developing their argument and incorporating evidence, they received full marks in this category. So far, so good!

Sophistication of Thought

Now, we know that this essay received a score of 5 out of 6 total points, and the place where the writer lost a point was on the basis of sophistication of thought, for which the writer received 0 out of 1 points. That's because this sample essay makes several generalizations and vague claims where it could have instead made specific claims that support a more balanced argument.

For example, in the following sentence from the 5th paragraph of the sample essay, the writer misses the opportunity to state specific possibilities that power companies should consider for wind energy . Instead, the writer is ambiguous and non-committal, saying, "As with most things, wind power has no easy answer. It is the responsibility of the companies building them to weigh the benefits and consequences."

If the writer of this essay was interested in trying to get that 6th point on the synthesis essay response, they could consider making more specific claims. For instance, they could state the specific benefits and consequences power companies should consider when deciding whether to establish a wind farm. These could include things like environmental impacts, economic impacts, or even population density!

Despite losing one point in the last category, this example synthesis essay is a strong one. It's well-developed, thoughtfully written, and advances an argument on the exam topic using evidence and support throughout.

body-number-four-post-it-note

4 Tips for How to Write a Synthesis Essay

AP Lang is a timed exam, so you have to pick and choose what you want to focus on in the limited time you're given to write the synthesis essay. Keep reading to get our expert advice on what you should focus on during your exam.

Tip 1: Read the Prompt First

It may sound obvious, but when you're pressed for time, it's easy to get flustered. Just remember: when it comes time to write the synthesis essay, read the prompt first !

Why is it so important to read the prompt before you read the sources? Because when you're aware of what kind of question you're trying to answer, you'll be able to read the sources more strategically. The prompt will help give you a sense of what claims, points, facts, or opinions to be looking for as you read the sources.

Reading the sources without having read the prompt first is kind of like trying to drive while wearing a blindfold: you can probably do it, but it's likely not going to end well!

Tip 2: Make Notes While You Read

During the 15-minute reading period at the beginning of the synthesis essay, you'll be reading through the sources as quickly as you can. After all, you're probably anxious to start writing!

While it's definitely important to make good use of your time, it's also important to read closely enough that you understand your sources. Careful reading will allow you to identify parts of the sources that will help you support your thesis statement in your essay, too.

As you read the sources, consider marking helpful passages with a star or check mark in the margins of the exam so you know which parts of the text to quickly re-read as you form your synthesis essay. You might also consider summing up the key points or position of each source in a sentence or a few words when you finish reading each source during the reading period. Doing so will help you know where each source stands on the topic given and help you pick the three (or more!) that will bolster your synthesis argument.

Tip 3: Start With the Thesis Statement

If you don't start your synthesis essay with a strong thesis statement, it's going to be tough to write an effective synthesis essay. As soon as you finish reading and annotating the provided sources, the thing you want to do next is write a strong thesis statement.

According to the CollegeBoard grading guidelines for the AP Lang synthesis essay, a strong thesis statement will respond to the prompt— not restate or rephrase the prompt. A good thesis will take a clear, defensible position on the topic presented in the prompt and the sources.

In other words, to write a solid thesis statement to guide the rest of your synthesis essay, you need to think about your position on the topic at hand and then make a claim about the topic based on your position. This position will either be defending, challenging, or qualifying the claim made in the essay's prompt.

The defensible position that you establish in your thesis statement will guide your argument in the rest of the essay, so it's important to do this first. Once you have a strong thesis statement, you can begin outlining your essay.

Tip 4: Focus on Your Commentary

Writing thoughtful, original commentary that explains your argument and your sources is important. In fact, doing this well will earn you four points (out of a total of six)!

AP Lang provides six to seven sources for you on the exam, and you'll be expected to incorporate quotations, paraphrases, or summaries from at least three of those sources into your synthesis essay and interpret that evidence for the reader.

While incorporating evidence is very important, in order to get the extra point for "sophistication of thought" on the synthesis essay, it's important to spend more time thinking about your commentary on the evidence you choose to incorporate. The commentary is your chance to show original thinking, strong rhetorical skills, and clearly explain how the evidence you've included supports the stance you laid out in your thesis statement.

To earn the 6th possible point on the synthesis essay, make sure your commentary demonstrates a nuanced understanding of the source material, explains this nuanced understanding, and places the evidence incorporated from the sources in conversation with each other. To do this, make sure you're avoiding vague language. Be specific when you can, and always tie your commentary back to your thesis!

body-person-arrows-next

What's Next?

There's a lot more to the AP Language exam than just the synthesis essay. Be sure to check out our expert guide to the entire exam , then learn more about the tricky multiple choice section .

Is the AP Lang exam hard...or is it easy? See how it stacks up to other AP tests on our list of the hardest AP exams .

Did you know there are technically two English AP exams? You can learn more about the second English AP test, the AP Literature exam, in this article . And if you're confused about whether you should take the AP Lang or AP Lit test , we can help you make that decision, too.

Want to improve your SAT score by 160 points or your ACT score by 4 points? We've written a guide for each test about the top 5 strategies you must be using to have a shot at improving your score. Download it for free now:

Get eBook: 5 Tips for 160+ Points

Ashley Sufflé Robinson has a Ph.D. in 19th Century English Literature. As a content writer for PrepScholar, Ashley is passionate about giving college-bound students the in-depth information they need to get into the school of their dreams.

Student and Parent Forum

Our new student and parent forum, at ExpertHub.PrepScholar.com , allow you to interact with your peers and the PrepScholar staff. See how other students and parents are navigating high school, college, and the college admissions process. Ask questions; get answers.

Join the Conversation

Ask a Question Below

Have any questions about this article or other topics? Ask below and we'll reply!

Improve With Our Famous Guides

  • For All Students

The 5 Strategies You Must Be Using to Improve 160+ SAT Points

How to Get a Perfect 1600, by a Perfect Scorer

Series: How to Get 800 on Each SAT Section:

Score 800 on SAT Math

Score 800 on SAT Reading

Score 800 on SAT Writing

Series: How to Get to 600 on Each SAT Section:

Score 600 on SAT Math

Score 600 on SAT Reading

Score 600 on SAT Writing

Free Complete Official SAT Practice Tests

What SAT Target Score Should You Be Aiming For?

15 Strategies to Improve Your SAT Essay

The 5 Strategies You Must Be Using to Improve 4+ ACT Points

How to Get a Perfect 36 ACT, by a Perfect Scorer

Series: How to Get 36 on Each ACT Section:

36 on ACT English

36 on ACT Math

36 on ACT Reading

36 on ACT Science

Series: How to Get to 24 on Each ACT Section:

24 on ACT English

24 on ACT Math

24 on ACT Reading

24 on ACT Science

What ACT target score should you be aiming for?

ACT Vocabulary You Must Know

ACT Writing: 15 Tips to Raise Your Essay Score

How to Get Into Harvard and the Ivy League

How to Get a Perfect 4.0 GPA

How to Write an Amazing College Essay

What Exactly Are Colleges Looking For?

Is the ACT easier than the SAT? A Comprehensive Guide

Should you retake your SAT or ACT?

When should you take the SAT or ACT?

Stay Informed

argumentative essay example ap lang

Get the latest articles and test prep tips!

Looking for Graduate School Test Prep?

Check out our top-rated graduate blogs here:

GRE Online Prep Blog

GMAT Online Prep Blog

TOEFL Online Prep Blog

Holly R. "I am absolutely overjoyed and cannot thank you enough for helping me!”

IMAGES

  1. AP Language and Composition argumentative essay

    argumentative essay example ap lang

  2. What Is an Argumentative Essay? Simple Examples To Guide You

    argumentative essay example ap lang

  3. AP Language Argument Prompt

    argumentative essay example ap lang

  4. How To Write An AP Lang Argumentative Essay?

    argumentative essay example ap lang

  5. 19+ Argument Essay Examples Ap Lang Background

    argumentative essay example ap lang

  6. AP Lang Argument Essay Outline by Walden's Wonderland

    argumentative essay example ap lang

VIDEO

  1. Argumentative Essay Topic Selection

  2. How to write #argumentative essays #essays @@AbiatalEnglish

  3. Argumentative Essay

  4. لقطة إنسانية من جمهور كوت ديفوار يساعد مشجع مغربي في ورطة صعبة في طريقه للعودة للمغرب💔

  5. How to identify an argumentative essay in CSS???

  6. Argumentative essay I Essay writing

COMMENTS

  1. How to Write the AP Lang Argument Essay (With Example)

    Her story "The Astronaut" won the 2018 Shirley Jackson Award for short fiction and received a "Distinguished Stories" mention in the 2019 Best American Short Stories anthology. Ap Lang Argumentative Essay - Expert advice on how to pen a winning essay + an AP Lang argument essay example to guide your writing.

  2. How to Write the AP Lang Argument Essay + Examples

    2. Pick one side of the argument, but acknowledge the other side. When you write the essay, it's best if you pick one side of the debate and stick with it for the entire essay. All your evidence should be in support of that one side. However, in your introductory paragraph, as you introduce the debate, be sure to mention any merit the ...

  3. AP English Language and Composition Past Exam Questions

    Download free-response questions from past exams along with scoring guidelines, sample responses from exam takers, and scoring distributions. If you are using assistive technology and need help accessing these PDFs in another format, contact Services for Students with Disabilities at 212-713-8333 or by email at [email protected]. Note ...

  4. PDF AP English Language and Composition

    AP ® English Language and Composition Sample Student Responses and Scoring Commentary Inside: Free Response Question 3 • Scoring Guideline • Student Samples ... Argument Essay 6 points . Many people spend long hours trying to achieve perfection in their personal or professional lives. Similarly, people often deman d perfection from

  5. PDF AP English Language and Composition

    While the response does not situate the argument within a broader context, the style is vivid and persuasive. Paragraph 2 provides an example of the response's persuasive style : "If reasoning and explanations were given when what I perceived as completely unfair and unnecessary rules were told to me, perhaps I would have been more receptive.

  6. PDF AP English Language and Composition Question 3: Argument (2019) Sample

    AP English Language and Composition Question 3: Argument (2019) Sample Student Responses 6 Sample E [1] ompetition is "overrated." The notion of motivation between peers has evolved into a source of unnecessary stress and even lack of morals. Whether it be in an academic environment or in the

  7. Crafting an Impressive Argumentative Essay for AP Lang

    Writing an argumentative essay for AP Language and Composition requires a strategic approach to effectively convey your perspective. Here's a guide to crafting an impressive argumentative essay: 1. Understand the Prompt: - Tip: Carefully read and analyze the prompt. Identify the key elements, including the task you are asked to perform and any ...

  8. How to Write the AP Lang Argument Essay + Examples

    What Is the AP Language Argument Essay? Tips for Writing the AP Language Argument Topic; AP English Country Line Essay Example; How Will AP Sheet Impact Mein College Chances? In 2023, over 550,148 students across the U.S. took to AP English English and Composition Exam, and 65.2% scored higher than a 3.

  9. How to Ace the AP Language Argument Essay

    Learn how to write the AP Language & Composition argument essay step by step in this tutorial! I'll show you the prompt from the 2018 exam and guide you thro...

  10. How to Writer the AP Lang Argument Essay + Examples

    Tips for Writing this AP Language Argument Essay; AP English Choice Quarrel Essay Examples; How Will AP Scores Impact My College Chances? Included 2023, over 550,148 apprentices across the U.S. took to AP English Language or Composition Exam, the 65.2% scored higher than a 3. The AP English Language Exam tests your ability to analyze a piece of ...

  11. Mastering The Argumentative Essay In AP Lang: Strategies And Examples

    Examples of Strong Argumentative Essays in AP Lang. To provide students with a clear understanding of what constitutes a strong argumentative essay in AP Lang, it is helpful to provide examples. One example is an essay that received a high score in the exam.

  12. How to Get a 6 on Argument FRQ in AP® English Language

    Pick an opinion and stick to it. Choose one side of the argument and one clear claim to support all the way through. Craft a thesis statement. Your thesis should be clear, concise, and introduce the content of your essay. Craft a chronological argument. Make an argument that builds on its prior points.

  13. PDF AP ENGLISH LANGUAGE AND COMPOSITION 2016 SCORING GUIDELINES

    clear prose, the essay was scored a 6. Sample: 3C Score: 4 . This essay inadequately argues a position on the extent to which Wilde's claims are valid. While the single piece of evidence presented (Animal Farm) has the potential to provide adequate support for the argument, the essay's explanation of how the evidence helps to develop the ...

  14. PDF AP English Language and Composition 2018 FRQ 3 Sample Student Responses

    AP English Language and Composition Question 3: Argument (2018) Sample Student Responses 3 Sample G [1] When I was nine year sold, my parents celebrated my birthday by taking me to a ropes course. As we waited with our group to climb poles, cross logs, and hang from ropes, I began to get cold feet. I've never done this before, I thought.

  15. PDF AP English Language and Composition

    AP® English Language and Composition 2022 Scoring Guidelines. Argument Essay 6 points . Colin Powell, a four-star general and former United States secretary of state, wrote in his 1995 autobiography: "[W]e do not have the luxury of collecting information indefinitely. At some point, before we can have every possible fact in hand, we have to ...

  16. AP Lang Argument Essay: Evidence

    Classical Argumentation. : Classical argumentation is an approach to persuasive writing that follows a specific structure known as the classical model. It consists of an introduction (exordium), background information (narratio), arguments (confirmatio), counterarguments (refutatio), and conclusion (peroratio).

  17. What's the best approach for the AP Lang argument essay?

    10. Write a strong conclusion: Sum up your main points, restate your thesis, and leave the reader with something thought-provoking or impactful. To prepare for the AP Lang argument essay, practice your writing skills regularly, read a variety of sources to broaden your knowledge, and analyze sample essays to understand what works and what doesn't.

  18. How to Write who AP Lang Argument Essay + Examples

    The AP English Language Exam as of 2023 is structured as next: Section 1: 45 multi-user choice questions to be completed within an hour. Like parcel counts for 45% of our score. This section requires students to analyze a piece of literature. The questions ask about its site and/or what could be edited at the passage.

  19. How to Write the AP Lang Synthesis Essay + Example

    The argumentative essay requires you to pick a perspective of a debate and create an argument based on the evidence provided. In this post, we will take a look at the AP Lang synthesis essay and discuss tips and tricks to master this part of the exam. We will also provide an example of a well-written essay for review.

  20. PDF ap06 english lang student samples

    Sample: 3B Score: 6. An adequate response, this essay asserts that knowing whom to trust is the key in this media-saturated world. The essay's three evidence paragraphs present examples of trustworthy and perhaps not-so-trustworthy people, including Rush Limbaugh, Tom Cruise, and retired CBS anchor Walter Cronkite.

  21. How to Write a Perfect Synthesis Essay for the AP Language Exam

    While there are multiple essay examples online, we've chosen one to take a closer look at. We're going to give you a brief analysis of one of these example student synthesis essays from the 2019 AP Lang Exam below! Example Synthesis Essay AP Lang Response. To get started, let's look at the official prompt for the 2019 synthesis essay:

  22. PDF AP Language Argument Prompts (some adaptions)

    In a well-organized essay, take a position on the relationship between certainty and doubt. Support your argument with appropriate evidence and examples. 2011-A The following passage is from Rights of Man, a book written by the pamphleteer Thomas Paine in 1791. Born in England, Paine was an intellectual, a revolutionary, and a supporter of American